s23-probability-theory/inputs/lecture_19.tex
2023-07-28 03:56:43 +02:00

251 lines
8.8 KiB
TeX

\lecture{19}{2023-06-22}{}
\subsection{Uniform Integrability}
\begin{example}
Let $\Omega = [0,1]$, $\cF = \cB$
and $\bP = \lambda \defon{[0,1]}$.
Consider $X_n \coloneqq n \One_{(0,\frac{1}{n}))}$.
We know that $X_n \xrightarrow{n \to \infty} 0$ a.s.,
however $\bE[X_n] = \bE[|X_n|] = 1$,
hence $X_n$ does not converge in $L^1(\bP)$.
Let $\mu_n(\cdot ) = \bP[X_n \in \cdot ]$.
Intuitively, for a series that converges in probability,
for $L^1$-convergence to hold we somehow need to make sure
that probability measures don't assign mass far away from $0$.
This will be made precise in the notion of uniform integrability.
\end{example}
\begin{goal}
We want to show that uniform integrability and convergence in probability
is equivalent to convergence in $L^1$.
\end{goal}
\begin{definition}
\label{def:ui}
A sequence of random variables $(X_n)_n$ is called \vocab{uniformly integrable} (UI),
if
\[\forall \epsilon > 0 .~\exists K > 0 .~ \forall n.~
\bE[|X_n| \One_{\{|X_n| > K\} }] < \epsilon.\]
Similarly, we define uniformly integrable for sets of random variables.
\end{definition}
\begin{example}
$X_n \coloneqq n \One_{(0,\frac{1}{n})}$ is not uniformly integrable.
\end{example}
There is no nice description of uniform integrability.
However, some subsets can be easily described, e.g.
\begin{fact}\label{lec19f1}
If $(X_n)_{n \ge 1}$ is a sequence bounded in $L^{1 + \delta}(\bP)$
for some $\delta > 0$ (i.e.~$\sup_n \bE[|X_n|^{1+\delta}] < \infty$),
then $(X_n)_n$ is uniformly integrable.
\end{fact}
\begin{proof}
Let $\epsilon > 0$.
Let $p \coloneqq 1 + \delta > 1$.
Choose $q$ such that $\frac{1}{p} + \frac{1}{q} = 1$.
Then
\begin{IEEEeqnarray*}{rCl}
\bE[|X_n| \One_{|X_n| > K}]
&\le& \bE[|X_n|^p]^{\frac{1}{p}} \bP[|X_n| > k]^{\frac{1}{q}},\\
\end{IEEEeqnarray*}
i.e.
\begin{IEEEeqnarray*}{rCl}
\sup_n\bE[|X_n| \One_{|X_n| > k}]
&\le& \underbrace{\sup_n\bE[|X_n|^p]^{\frac{1}{p}}}_{< \infty}
\sup_n \underbrace{\bP[|X_n| > K]^{\frac{1}{q}}}_%
{\le K^{-\frac{1}{q}} \bE[|X_n|]^{\frac{1}{q}}}\\
\end{IEEEeqnarray*}
where we have applied \yaref{thm:markov}.
Since $\sup_n \bE[|X_n|^{1+\delta}] < \infty$,
we have that $\sup_n \bE[|X_n|] < \infty$ by \yaref{jensen}.
Hence for $K$ large enough the relevant term is less than $\epsilon$.
\end{proof}
\begin{fact}\label{lec19f2}
If $(X_n)_n$ is uniformly integrable,
then $(X_n)_n$ is bounded in $L^1$.
\end{fact}
\begin{proof}
Take some $\epsilon > 0$ and $K$ such that
$\sup_n\bE[|X_n| \One_{|X_n| > K}] < \epsilon$.
Then $\sup_n\|X_n\|_{L^1} \le K + \epsilon$.
\end{proof}
\begin{fact}\label{lec19f3}
Suppose $Y \in L^1(\bP)$ and $\sup_n |X_n(\cdot )| \le Y(\cdot )$.
Then $(X_n)_n$ is uniformly integrable.
\end{fact}
\begin{fact}\label{lec19f4}
Let $X \in L^1(\bP)$.
\begin{enumerate}[(a)]
\item $\forall \epsilon > 0 .~ \exists \delta > 0 .~\forall F \in \cF .~ \bP(F) < \delta \implies\int_F |X| \dif \bP < \epsilon$.
\item $\forall \epsilon > 0 .~ \exists k \in (0,\infty) .~ \int_{|X| > k} | X| \dif \bP < \epsilon$.
\end{enumerate}
\end{fact}
\begin{proof}
\begin{enumerate}[(a)]
\item Suppose not. Then for $\delta = 1, \frac{1}{2}, \frac{1}{2^2}, \ldots$
there exists $F_n$ such that $\bP(F_n) <\frac{1}{2^n}$
but $\int_{F_n} |X| \dif \bP \ge \epsilon$.
Since $\sum_{n} \bP(F_n) < \infty$,
by \yaref{thm:borelcantelli},
\[\bP[\underbrace{\limsup_n F_n}_{\text{\reflectbox{$\coloneqq$}}F}] = 0.\]
We have
\begin{IEEEeqnarray*}{rCl}
\int_F | X| \dif \bP &=& \int |X| \One_F \dif \bP\\
&=& \int \limsup_n (|X| \One_{F_n}) \dif \bP\\
&\overset{\text{Reverse Fatou}}{\ge }&
\limsup_n \int |X| \One_{F_n} \dif \bP\\
&\ge & \epsilon
\end{IEEEeqnarray*}
where the assumption that $X$ is in $L^1$ was used to apply
the reverse of Fatou's lemma. % TODO reverse fatou
This yields a contradiction since $\bP(F) = 0$.
\item We want to apply part (a) to $F = \{ |X| > k\}$.
By \yaref{thm:markov}, $\bP(F) \le \frac{1}{k} \bE[|X|]$.
Since $\bE[|X|] < \infty$, we can choose $k$ large enough
to get $\bP(F) \le \delta$.
\end{enumerate}
\end{proof}
\begin{refproof}{lec19f3}
Fix $\epsilon > 0$.
We have
\[
\bE[|X_n| \One_{|X_n| > k}] \le \bE[|Y| \One_{|Y| > k}] < \epsilon
\]
for $k$ large enough by \yaref{lec19f4} (b).
\end{refproof}
\begin{fact}\label{lec19f5}
Let $X \in L^1(\bP)$.
Then $\bF \coloneqq \{ \bE[X | \cG] : \cG \subseteq \cF \text{ sub-$\sigma$-algebra}\}$ is uniformly integrable.
\end{fact}
\begin{proof}
Fix $\epsilon > 0$.
Choose $\delta > 0$ such that
\begin{equation}
\forall F \in \cF.~ \bP(F) < \delta \implies \bE[|X| \One_F] <\epsilon.
\label{lec19eqstar}
\end{equation}
Let $Y = \bE[X | \cG]$ for some sub-$\sigma$-algebra $\cG$.
Then, by \yaref{cjensen}, $|Y| \le \bE[ |X| | \cG]$.
Hence $\bE[|Y|] \le \bE[|X|]$.
By \yaref{thm:markov},
it follows that $\bP[|Y| > k] < \delta$
for $k > \frac{\bE[|X|]}{\delta}$.
Note that $\{|Y| > k\} \in \cG$.
We have
\begin{IEEEeqnarray*}{rCl}
\bE[|Y| \One_{\{|Y| > k\} }] &<& \epsilon
\end{IEEEeqnarray*}
by \eqref{lec19eqstar}, since $\bP[|Y| > k] < \delta$.
\end{proof}
\begin{theorem}
\label{thm:l1iffuip}
Assume that $X_n \in L^1$ for all $n$ and $X \in L^1$.
Then the following are equivalent:
\begin{enumerate}[(1)]
\item $X_n \to X$ in $L^1$.
\item $(X_n)_n$ is uniformly integrable and $X_n \to X$ in probability.
\end{enumerate}
\end{theorem}
\begin{proof}
(2) $\implies$ (1)
Define
\begin{IEEEeqnarray*}{rCl}
\phi(x) &\coloneqq & \begin{cases}
-k, & x \le -k\\
x, & x \in (-k,k)\\
k, & x \ge k.
\end{cases}
\end{IEEEeqnarray*}
$\phi$ is $1$-Lipschitz. % TODO
We have
\begin{IEEEeqnarray*}{rCl}
\int |X_n - X| \dif \bP
&\le & \int |X_n - \phi(X_n)| \dif \bP
+ \int |\phi(X) - X| \dif \bP
+ \int |\phi(X_n) - \phi(X)| \dif \bP\\
\end{IEEEeqnarray*}
We have $\int_{|X_n| > k} \underbrace{|X_n - \phi(X_n)|}_{\le |X_n| + | \phi(X_n)| \le 2 |X_n|} \dif \bP\le \epsilon$ by uniform integrability and
\yaref{lec19f4} part (b).
Similarly $\int_{|X| > k} |X - \phi(X)| \dif \bP < \epsilon$.
Since $\phi$ is Lipschitz,
$ X_n \xrightarrow{\bP} X \implies \phi(X_n) \xrightarrow{\bP} \phi(X)$.
By the \yaref{thm:boundedconvergence}
$|\phi(X_n)| \le k \implies \int | \phi(X_n) - \phi(X)| \dif \bP \to 0$.
(1) $\implies$ (2)
$X_n \xrightarrow{L^1} X \implies X_n \xrightarrow{\bP} X$
by \yaref{thm:markov} (see \yaref{claim:convimpll1p}).
Fix $\epsilon > 0$.
We have
\begin{IEEEeqnarray*}{rCl}
\bE[|X_n|] &=& \bE[|X_n - X + X|]\\
&\le & \epsilon + \bE[|X|]\\
&<& \delta k
\end{IEEEeqnarray*}
for all $\delta > 0$ and suitable $k$.
Hence $\bP[|X_n| > k] < \delta$ by \yaref{thm:markov}.
Then by \yaref{lec19f4} part (a) it follows that
\[
\int_{|X_n| > k} |X_n| \dif \bP \le \underbrace{\int |X - X_n| \dif \bP}_{< \epsilon} + \int_{|X_n| > k} |X| \dif \bP \le 2 \epsilon.
\]
\end{proof}
\subsection{Martingale Convergence Theorems in \texorpdfstring{$L^p, p \ge 1$}{$Lp, p >= 1$}}
Let $(\Omega, \cF, \bP)$ as always and let $(\cF_n)_n$ always be a filtration.
\begin{fact}\label{lec19f6}
Suppose that $X \in L^p$ for some $p \ge 1$.
Then $(\bE[X | \cF_n])_n$ is an $\cF_n$-martingale.
\end{fact}
\begin{proof}
It is clear that $(\bE[X | \cF_n])_n$ is adapted to $(\cF_n)_n$.
Let $X_n \coloneqq \bE[X | \cF_n]$.
Consider
\begin{IEEEeqnarray*}{rCl}
\bE[X_n - X_{n-1} | \cF_{n-1}]
&=& \bE[\bE[X | \cF_n] - \bE[X | \cF_{n-1}] | \cF_{n-1}]\\
&=& \bE[X | \cF_{n-1}] - \bE[X | \cF_{n-1}]\\
&=& 0.
\end{IEEEeqnarray*}
\end{proof}
\begin{theorem}
\label{ceismartingale}
Let $X \in L^p$ for some $p \ge 1$
and $\bigcup_n \cF_n \to \cF \supseteq \sigma(X)$.
Then $X_n \coloneqq \bE[X | \cF_n]$ defines a martingale which converges
to $X$ in $L^p$.
\end{theorem}
% \todo{Proof ?}
\begin{theorem}
\label{martingaleisce}
Let $p > 1$.
Let $(X_n)_n$ be a martingale bounded in $L^p$.
Then there exists a random variable $X \in L^p$, such that
$X_n = \bE[X | \cF_n]$ for all $n$.
In particular, $X_n \xrightarrow{L^p} X$.
\end{theorem}